Find the value of $alpha$ given values $sin(alpha + frac{pi}{6})$ and $tan(alpha)$ The Next...

Anatomically Correct Mesopelagic Aves

Does it take more energy to get to Venus or to Mars?

Visit to the USA with ESTA approved before trip to Iran

How easy is it to start Magic from scratch?

How should I support this large drywall patch?

Natural language into sentence logic

Grabbing quick drinks

Increase performance creating Mandelbrot set in python

How long to clear the 'suck zone' of a turbofan after start is initiated?

How to write papers efficiently when English isn't my first language?

Should I tutor a student who I know has cheated on their homework?

Are there languages with no euphemisms?

How to get regions to plot as graphics

Whats the best way to handle refactoring a big file?

Can a caster that cast Polymorph on themselves stop concentrating at any point even if their Int is low?

When Does an Atlas Uniquely Define a Manifold?

Go Pregnant or Go Home

Was a professor correct to chastise me for writing "Prof. X" rather than "Professor X"?

What is the difference between "behavior" and "behaviour"?

Text adventure game code

Can I equip Skullclamp on a creature I am sacrificing?

Why did we only see the N-1 starfighters in one film?

Is a stroke of luck acceptable after a series of unfavorable events?

Why didn't Khan get resurrected in the Genesis Explosion?



Find the value of $alpha$ given values $sin(alpha + frac{pi}{6})$ and $tan(alpha)$



The Next CEO of Stack OverflowFind $sin theta$ and $cos theta$ given $tan 2theta$Trigonometry question, find the value if $cos(x) = frac{5}{13}$ and $cos(y) = -frac{5}{13}$finding the slope of a line connecting two given Terminal pointsEasy question Find $sin 2x$, $cos 2x$, and $tan 2x$Prove identity: $frac{1+sinalpha-cosalpha}{1+sinalpha+cosalpha}=tanfrac{alpha}{2}$Use appropriate identities to find sin (theta) and cos(theta) if tan(theta) = 0.4 and the terminal side of (theta) lies in quadrant III.Find the value of $tan A + tan B$, given values of $frac{sin (A)}{sin (B)}$ and $frac{cos (A)}{cos (B)}$Find the exact value of $sin(pi+alpha)+cos(frac{3pi}{2}+alpha)+tan(-frac{pi}{2}+alpha)$To use trigonometric identities to find the value of $tanalpha$Trigonometric Identities: Given $tan(2a)=2$ and $frac{3pi}{2}<a<2pi$ find value of $tan(a)$












1












$begingroup$


Find $alpha$, given:




  • $sin(alpha + frac{pi}{6})$


  • $tan(alpha)=-5$


  • And terminal side is in Quadrant II



I only got as far as figuring out that the coordinate for the point in QII is $(-1,5)$ and the length of that segment is $sqrt{26}$. But I'm stuck in getting $alpha$.










share|cite|improve this question











$endgroup$












  • $begingroup$
    ...what value is the sine of alpha plus a sixth of pi (the first one)?
    $endgroup$
    – Parcly Taxel
    Aug 24 '16 at 14:10










  • $begingroup$
    that is the only information given.
    $endgroup$
    – Paul
    Aug 24 '16 at 14:46










  • $begingroup$
    Isn't $alpha$ simply $tan^{-1}(-5)$?
    $endgroup$
    – Parcly Taxel
    Aug 24 '16 at 14:47










  • $begingroup$
    That's what I was thinking. I'll ask my professor. Thanks!
    $endgroup$
    – Paul
    Aug 24 '16 at 15:07
















1












$begingroup$


Find $alpha$, given:




  • $sin(alpha + frac{pi}{6})$


  • $tan(alpha)=-5$


  • And terminal side is in Quadrant II



I only got as far as figuring out that the coordinate for the point in QII is $(-1,5)$ and the length of that segment is $sqrt{26}$. But I'm stuck in getting $alpha$.










share|cite|improve this question











$endgroup$












  • $begingroup$
    ...what value is the sine of alpha plus a sixth of pi (the first one)?
    $endgroup$
    – Parcly Taxel
    Aug 24 '16 at 14:10










  • $begingroup$
    that is the only information given.
    $endgroup$
    – Paul
    Aug 24 '16 at 14:46










  • $begingroup$
    Isn't $alpha$ simply $tan^{-1}(-5)$?
    $endgroup$
    – Parcly Taxel
    Aug 24 '16 at 14:47










  • $begingroup$
    That's what I was thinking. I'll ask my professor. Thanks!
    $endgroup$
    – Paul
    Aug 24 '16 at 15:07














1












1








1


0



$begingroup$


Find $alpha$, given:




  • $sin(alpha + frac{pi}{6})$


  • $tan(alpha)=-5$


  • And terminal side is in Quadrant II



I only got as far as figuring out that the coordinate for the point in QII is $(-1,5)$ and the length of that segment is $sqrt{26}$. But I'm stuck in getting $alpha$.










share|cite|improve this question











$endgroup$




Find $alpha$, given:




  • $sin(alpha + frac{pi}{6})$


  • $tan(alpha)=-5$


  • And terminal side is in Quadrant II



I only got as far as figuring out that the coordinate for the point in QII is $(-1,5)$ and the length of that segment is $sqrt{26}$. But I'm stuck in getting $alpha$.







trigonometry






share|cite|improve this question















share|cite|improve this question













share|cite|improve this question




share|cite|improve this question








edited Mar 16 at 7:40









Robert Howard

2,2783935




2,2783935










asked Aug 24 '16 at 13:23









PaulPaul

1091




1091












  • $begingroup$
    ...what value is the sine of alpha plus a sixth of pi (the first one)?
    $endgroup$
    – Parcly Taxel
    Aug 24 '16 at 14:10










  • $begingroup$
    that is the only information given.
    $endgroup$
    – Paul
    Aug 24 '16 at 14:46










  • $begingroup$
    Isn't $alpha$ simply $tan^{-1}(-5)$?
    $endgroup$
    – Parcly Taxel
    Aug 24 '16 at 14:47










  • $begingroup$
    That's what I was thinking. I'll ask my professor. Thanks!
    $endgroup$
    – Paul
    Aug 24 '16 at 15:07


















  • $begingroup$
    ...what value is the sine of alpha plus a sixth of pi (the first one)?
    $endgroup$
    – Parcly Taxel
    Aug 24 '16 at 14:10










  • $begingroup$
    that is the only information given.
    $endgroup$
    – Paul
    Aug 24 '16 at 14:46










  • $begingroup$
    Isn't $alpha$ simply $tan^{-1}(-5)$?
    $endgroup$
    – Parcly Taxel
    Aug 24 '16 at 14:47










  • $begingroup$
    That's what I was thinking. I'll ask my professor. Thanks!
    $endgroup$
    – Paul
    Aug 24 '16 at 15:07
















$begingroup$
...what value is the sine of alpha plus a sixth of pi (the first one)?
$endgroup$
– Parcly Taxel
Aug 24 '16 at 14:10




$begingroup$
...what value is the sine of alpha plus a sixth of pi (the first one)?
$endgroup$
– Parcly Taxel
Aug 24 '16 at 14:10












$begingroup$
that is the only information given.
$endgroup$
– Paul
Aug 24 '16 at 14:46




$begingroup$
that is the only information given.
$endgroup$
– Paul
Aug 24 '16 at 14:46












$begingroup$
Isn't $alpha$ simply $tan^{-1}(-5)$?
$endgroup$
– Parcly Taxel
Aug 24 '16 at 14:47




$begingroup$
Isn't $alpha$ simply $tan^{-1}(-5)$?
$endgroup$
– Parcly Taxel
Aug 24 '16 at 14:47












$begingroup$
That's what I was thinking. I'll ask my professor. Thanks!
$endgroup$
– Paul
Aug 24 '16 at 15:07




$begingroup$
That's what I was thinking. I'll ask my professor. Thanks!
$endgroup$
– Paul
Aug 24 '16 at 15:07










1 Answer
1






active

oldest

votes


















1












$begingroup$

Unfortunately, it's not quite as simple as taking the arctangent of $tan(alpha)$. Were your point in Quadrant I, it would be that simple, but that's not the case. On the bright side, it's not a whole lot more complicated than that.



Well, maybe a little more complicated.





The answer you're looking for is $$alpha=arctan(-5)+pi$$ I could just leave it at that and say that the $+hspace{1mm}pi$ comes from some magical property of the arctangent function, but that's no fun. The fun part is why we need to add some integer multiple to the output of the arctangent function to get the correct answer, and why that integer changes depending on what quadrant the angle you want to find happens to be in.



Call that integer $n$. In Quadrant I, $n=0$. In Quadrants II and III, $n=1$, and in Quadrant IV, $n=2$ (or $0$). To get a feeling for why this is, let's look at the graph of $y=arctan(tan(x))$. I've added dashed lines at odd integer multiples of $frac{pi}{2}$, and the regions between them correspond to $n=-2$, $n=-1$, $n=0$, $n=1$, and $n=2$, respectively, from left to right.



$hspace{2.25cm}$



Seeing as $arctan$ and $tan$ are inverse functions, we would expect the graph of $y=arctan(tan(x))$ to look like the graph of $y=x$. That's clearly not the case; instead, it looks like the graph of $y=x$ has been split up into $frac{pi}{2}timesfrac{pi}{2}$ chunks that are then all lined up on the $x$-axis.



The reason why the above graph looks the way it does has to do with how we define the arctangent function. The tangent function isn't inherently one-to-one, so in order to define its inverse, we need to restrict its domain to $left(-frac{pi}{2},frac{pi}{2}right)$, which means that the range of the arctangent function is also $left(-frac{pi}{2},frac{pi}{2}right)$. Unfortunately, angles greater in magnitude than $frac{pi}{2}$ do exist. So what's going to happen when you want the arctangent function to produce one of those angles? It'll produce some number in its range that is an integer multiple of $pi$ away from the angle you're looking for.



For example, $arctan(tan(2pi))$ should obviously return $2pi$, right? Instead, the result is $0$, which, by some stroke of sheer mathematical luck, happens to be an integer multiple of $pi$ away from $2pi$ (joking a little here). For a more interesting example, $arctanleft(tanleft(frac{7pi}{4}right)right)$ returns $-frac{pi}{4}$. Since $frac{7pi}{4}$ and $-frac{pi}{4}$ are equivalent on the unit circle, this might be fine in some settings, but in others, all you need to do is add $2pi$ to get back to $frac{7pi}{4}$.



Finally, notice that the act of adding integer multiples of $pi$ to your answers here is like moving the line segments in the graph I included up or down by integer multiples of $pi$ to reconstruct the graph of $y=x$.






share|cite|improve this answer











$endgroup$














    Your Answer





    StackExchange.ifUsing("editor", function () {
    return StackExchange.using("mathjaxEditing", function () {
    StackExchange.MarkdownEditor.creationCallbacks.add(function (editor, postfix) {
    StackExchange.mathjaxEditing.prepareWmdForMathJax(editor, postfix, [["$", "$"], ["\\(","\\)"]]);
    });
    });
    }, "mathjax-editing");

    StackExchange.ready(function() {
    var channelOptions = {
    tags: "".split(" "),
    id: "69"
    };
    initTagRenderer("".split(" "), "".split(" "), channelOptions);

    StackExchange.using("externalEditor", function() {
    // Have to fire editor after snippets, if snippets enabled
    if (StackExchange.settings.snippets.snippetsEnabled) {
    StackExchange.using("snippets", function() {
    createEditor();
    });
    }
    else {
    createEditor();
    }
    });

    function createEditor() {
    StackExchange.prepareEditor({
    heartbeatType: 'answer',
    autoActivateHeartbeat: false,
    convertImagesToLinks: true,
    noModals: true,
    showLowRepImageUploadWarning: true,
    reputationToPostImages: 10,
    bindNavPrevention: true,
    postfix: "",
    imageUploader: {
    brandingHtml: "Powered by u003ca class="icon-imgur-white" href="https://imgur.com/"u003eu003c/au003e",
    contentPolicyHtml: "User contributions licensed under u003ca href="https://creativecommons.org/licenses/by-sa/3.0/"u003ecc by-sa 3.0 with attribution requiredu003c/au003e u003ca href="https://stackoverflow.com/legal/content-policy"u003e(content policy)u003c/au003e",
    allowUrls: true
    },
    noCode: true, onDemand: true,
    discardSelector: ".discard-answer"
    ,immediatelyShowMarkdownHelp:true
    });


    }
    });














    draft saved

    draft discarded


















    StackExchange.ready(
    function () {
    StackExchange.openid.initPostLogin('.new-post-login', 'https%3a%2f%2fmath.stackexchange.com%2fquestions%2f1902120%2ffind-the-value-of-alpha-given-values-sin-alpha-frac-pi6-and-tan%23new-answer', 'question_page');
    }
    );

    Post as a guest















    Required, but never shown

























    1 Answer
    1






    active

    oldest

    votes








    1 Answer
    1






    active

    oldest

    votes









    active

    oldest

    votes






    active

    oldest

    votes









    1












    $begingroup$

    Unfortunately, it's not quite as simple as taking the arctangent of $tan(alpha)$. Were your point in Quadrant I, it would be that simple, but that's not the case. On the bright side, it's not a whole lot more complicated than that.



    Well, maybe a little more complicated.





    The answer you're looking for is $$alpha=arctan(-5)+pi$$ I could just leave it at that and say that the $+hspace{1mm}pi$ comes from some magical property of the arctangent function, but that's no fun. The fun part is why we need to add some integer multiple to the output of the arctangent function to get the correct answer, and why that integer changes depending on what quadrant the angle you want to find happens to be in.



    Call that integer $n$. In Quadrant I, $n=0$. In Quadrants II and III, $n=1$, and in Quadrant IV, $n=2$ (or $0$). To get a feeling for why this is, let's look at the graph of $y=arctan(tan(x))$. I've added dashed lines at odd integer multiples of $frac{pi}{2}$, and the regions between them correspond to $n=-2$, $n=-1$, $n=0$, $n=1$, and $n=2$, respectively, from left to right.



    $hspace{2.25cm}$



    Seeing as $arctan$ and $tan$ are inverse functions, we would expect the graph of $y=arctan(tan(x))$ to look like the graph of $y=x$. That's clearly not the case; instead, it looks like the graph of $y=x$ has been split up into $frac{pi}{2}timesfrac{pi}{2}$ chunks that are then all lined up on the $x$-axis.



    The reason why the above graph looks the way it does has to do with how we define the arctangent function. The tangent function isn't inherently one-to-one, so in order to define its inverse, we need to restrict its domain to $left(-frac{pi}{2},frac{pi}{2}right)$, which means that the range of the arctangent function is also $left(-frac{pi}{2},frac{pi}{2}right)$. Unfortunately, angles greater in magnitude than $frac{pi}{2}$ do exist. So what's going to happen when you want the arctangent function to produce one of those angles? It'll produce some number in its range that is an integer multiple of $pi$ away from the angle you're looking for.



    For example, $arctan(tan(2pi))$ should obviously return $2pi$, right? Instead, the result is $0$, which, by some stroke of sheer mathematical luck, happens to be an integer multiple of $pi$ away from $2pi$ (joking a little here). For a more interesting example, $arctanleft(tanleft(frac{7pi}{4}right)right)$ returns $-frac{pi}{4}$. Since $frac{7pi}{4}$ and $-frac{pi}{4}$ are equivalent on the unit circle, this might be fine in some settings, but in others, all you need to do is add $2pi$ to get back to $frac{7pi}{4}$.



    Finally, notice that the act of adding integer multiples of $pi$ to your answers here is like moving the line segments in the graph I included up or down by integer multiples of $pi$ to reconstruct the graph of $y=x$.






    share|cite|improve this answer











    $endgroup$


















      1












      $begingroup$

      Unfortunately, it's not quite as simple as taking the arctangent of $tan(alpha)$. Were your point in Quadrant I, it would be that simple, but that's not the case. On the bright side, it's not a whole lot more complicated than that.



      Well, maybe a little more complicated.





      The answer you're looking for is $$alpha=arctan(-5)+pi$$ I could just leave it at that and say that the $+hspace{1mm}pi$ comes from some magical property of the arctangent function, but that's no fun. The fun part is why we need to add some integer multiple to the output of the arctangent function to get the correct answer, and why that integer changes depending on what quadrant the angle you want to find happens to be in.



      Call that integer $n$. In Quadrant I, $n=0$. In Quadrants II and III, $n=1$, and in Quadrant IV, $n=2$ (or $0$). To get a feeling for why this is, let's look at the graph of $y=arctan(tan(x))$. I've added dashed lines at odd integer multiples of $frac{pi}{2}$, and the regions between them correspond to $n=-2$, $n=-1$, $n=0$, $n=1$, and $n=2$, respectively, from left to right.



      $hspace{2.25cm}$



      Seeing as $arctan$ and $tan$ are inverse functions, we would expect the graph of $y=arctan(tan(x))$ to look like the graph of $y=x$. That's clearly not the case; instead, it looks like the graph of $y=x$ has been split up into $frac{pi}{2}timesfrac{pi}{2}$ chunks that are then all lined up on the $x$-axis.



      The reason why the above graph looks the way it does has to do with how we define the arctangent function. The tangent function isn't inherently one-to-one, so in order to define its inverse, we need to restrict its domain to $left(-frac{pi}{2},frac{pi}{2}right)$, which means that the range of the arctangent function is also $left(-frac{pi}{2},frac{pi}{2}right)$. Unfortunately, angles greater in magnitude than $frac{pi}{2}$ do exist. So what's going to happen when you want the arctangent function to produce one of those angles? It'll produce some number in its range that is an integer multiple of $pi$ away from the angle you're looking for.



      For example, $arctan(tan(2pi))$ should obviously return $2pi$, right? Instead, the result is $0$, which, by some stroke of sheer mathematical luck, happens to be an integer multiple of $pi$ away from $2pi$ (joking a little here). For a more interesting example, $arctanleft(tanleft(frac{7pi}{4}right)right)$ returns $-frac{pi}{4}$. Since $frac{7pi}{4}$ and $-frac{pi}{4}$ are equivalent on the unit circle, this might be fine in some settings, but in others, all you need to do is add $2pi$ to get back to $frac{7pi}{4}$.



      Finally, notice that the act of adding integer multiples of $pi$ to your answers here is like moving the line segments in the graph I included up or down by integer multiples of $pi$ to reconstruct the graph of $y=x$.






      share|cite|improve this answer











      $endgroup$
















        1












        1








        1





        $begingroup$

        Unfortunately, it's not quite as simple as taking the arctangent of $tan(alpha)$. Were your point in Quadrant I, it would be that simple, but that's not the case. On the bright side, it's not a whole lot more complicated than that.



        Well, maybe a little more complicated.





        The answer you're looking for is $$alpha=arctan(-5)+pi$$ I could just leave it at that and say that the $+hspace{1mm}pi$ comes from some magical property of the arctangent function, but that's no fun. The fun part is why we need to add some integer multiple to the output of the arctangent function to get the correct answer, and why that integer changes depending on what quadrant the angle you want to find happens to be in.



        Call that integer $n$. In Quadrant I, $n=0$. In Quadrants II and III, $n=1$, and in Quadrant IV, $n=2$ (or $0$). To get a feeling for why this is, let's look at the graph of $y=arctan(tan(x))$. I've added dashed lines at odd integer multiples of $frac{pi}{2}$, and the regions between them correspond to $n=-2$, $n=-1$, $n=0$, $n=1$, and $n=2$, respectively, from left to right.



        $hspace{2.25cm}$



        Seeing as $arctan$ and $tan$ are inverse functions, we would expect the graph of $y=arctan(tan(x))$ to look like the graph of $y=x$. That's clearly not the case; instead, it looks like the graph of $y=x$ has been split up into $frac{pi}{2}timesfrac{pi}{2}$ chunks that are then all lined up on the $x$-axis.



        The reason why the above graph looks the way it does has to do with how we define the arctangent function. The tangent function isn't inherently one-to-one, so in order to define its inverse, we need to restrict its domain to $left(-frac{pi}{2},frac{pi}{2}right)$, which means that the range of the arctangent function is also $left(-frac{pi}{2},frac{pi}{2}right)$. Unfortunately, angles greater in magnitude than $frac{pi}{2}$ do exist. So what's going to happen when you want the arctangent function to produce one of those angles? It'll produce some number in its range that is an integer multiple of $pi$ away from the angle you're looking for.



        For example, $arctan(tan(2pi))$ should obviously return $2pi$, right? Instead, the result is $0$, which, by some stroke of sheer mathematical luck, happens to be an integer multiple of $pi$ away from $2pi$ (joking a little here). For a more interesting example, $arctanleft(tanleft(frac{7pi}{4}right)right)$ returns $-frac{pi}{4}$. Since $frac{7pi}{4}$ and $-frac{pi}{4}$ are equivalent on the unit circle, this might be fine in some settings, but in others, all you need to do is add $2pi$ to get back to $frac{7pi}{4}$.



        Finally, notice that the act of adding integer multiples of $pi$ to your answers here is like moving the line segments in the graph I included up or down by integer multiples of $pi$ to reconstruct the graph of $y=x$.






        share|cite|improve this answer











        $endgroup$



        Unfortunately, it's not quite as simple as taking the arctangent of $tan(alpha)$. Were your point in Quadrant I, it would be that simple, but that's not the case. On the bright side, it's not a whole lot more complicated than that.



        Well, maybe a little more complicated.





        The answer you're looking for is $$alpha=arctan(-5)+pi$$ I could just leave it at that and say that the $+hspace{1mm}pi$ comes from some magical property of the arctangent function, but that's no fun. The fun part is why we need to add some integer multiple to the output of the arctangent function to get the correct answer, and why that integer changes depending on what quadrant the angle you want to find happens to be in.



        Call that integer $n$. In Quadrant I, $n=0$. In Quadrants II and III, $n=1$, and in Quadrant IV, $n=2$ (or $0$). To get a feeling for why this is, let's look at the graph of $y=arctan(tan(x))$. I've added dashed lines at odd integer multiples of $frac{pi}{2}$, and the regions between them correspond to $n=-2$, $n=-1$, $n=0$, $n=1$, and $n=2$, respectively, from left to right.



        $hspace{2.25cm}$



        Seeing as $arctan$ and $tan$ are inverse functions, we would expect the graph of $y=arctan(tan(x))$ to look like the graph of $y=x$. That's clearly not the case; instead, it looks like the graph of $y=x$ has been split up into $frac{pi}{2}timesfrac{pi}{2}$ chunks that are then all lined up on the $x$-axis.



        The reason why the above graph looks the way it does has to do with how we define the arctangent function. The tangent function isn't inherently one-to-one, so in order to define its inverse, we need to restrict its domain to $left(-frac{pi}{2},frac{pi}{2}right)$, which means that the range of the arctangent function is also $left(-frac{pi}{2},frac{pi}{2}right)$. Unfortunately, angles greater in magnitude than $frac{pi}{2}$ do exist. So what's going to happen when you want the arctangent function to produce one of those angles? It'll produce some number in its range that is an integer multiple of $pi$ away from the angle you're looking for.



        For example, $arctan(tan(2pi))$ should obviously return $2pi$, right? Instead, the result is $0$, which, by some stroke of sheer mathematical luck, happens to be an integer multiple of $pi$ away from $2pi$ (joking a little here). For a more interesting example, $arctanleft(tanleft(frac{7pi}{4}right)right)$ returns $-frac{pi}{4}$. Since $frac{7pi}{4}$ and $-frac{pi}{4}$ are equivalent on the unit circle, this might be fine in some settings, but in others, all you need to do is add $2pi$ to get back to $frac{7pi}{4}$.



        Finally, notice that the act of adding integer multiples of $pi$ to your answers here is like moving the line segments in the graph I included up or down by integer multiples of $pi$ to reconstruct the graph of $y=x$.







        share|cite|improve this answer














        share|cite|improve this answer



        share|cite|improve this answer








        edited Mar 16 at 18:15

























        answered Mar 16 at 7:39









        Robert HowardRobert Howard

        2,2783935




        2,2783935






























            draft saved

            draft discarded




















































            Thanks for contributing an answer to Mathematics Stack Exchange!


            • Please be sure to answer the question. Provide details and share your research!

            But avoid



            • Asking for help, clarification, or responding to other answers.

            • Making statements based on opinion; back them up with references or personal experience.


            Use MathJax to format equations. MathJax reference.


            To learn more, see our tips on writing great answers.




            draft saved


            draft discarded














            StackExchange.ready(
            function () {
            StackExchange.openid.initPostLogin('.new-post-login', 'https%3a%2f%2fmath.stackexchange.com%2fquestions%2f1902120%2ffind-the-value-of-alpha-given-values-sin-alpha-frac-pi6-and-tan%23new-answer', 'question_page');
            }
            );

            Post as a guest















            Required, but never shown





















































            Required, but never shown














            Required, but never shown












            Required, but never shown







            Required, but never shown

































            Required, but never shown














            Required, but never shown












            Required, but never shown







            Required, but never shown







            Popular posts from this blog

            Magento 2 - Add success message with knockout Planned maintenance scheduled April 23, 2019 at 23:30 UTC (7:30pm US/Eastern) Announcing the arrival of Valued Associate #679: Cesar Manara Unicorn Meta Zoo #1: Why another podcast?Success / Error message on ajax request$.widget is not a function when loading a homepage after add custom jQuery on custom themeHow can bind jQuery to current document in Magento 2 When template load by ajaxRedirect page using plugin in Magento 2Magento 2 - Update quantity and totals of cart page without page reload?Magento 2: Quote data not loaded on knockout checkoutMagento 2 : I need to change add to cart success message after adding product into cart through pluginMagento 2.2.5 How to add additional products to cart from new checkout step?Magento 2 Add error/success message with knockoutCan't validate Post Code on checkout page

            Fil:Tokke komm.svg

            Where did Arya get these scars? Unicorn Meta Zoo #1: Why another podcast? Announcing the arrival of Valued Associate #679: Cesar Manara Favourite questions and answers from the 1st quarter of 2019Why did Arya refuse to end it?Has the pronunciation of Arya Stark's name changed?Has Arya forgiven people?Why did Arya Stark lose her vision?Why can Arya still use the faces?Has the Narrow Sea become narrower?Does Arya Stark know how to make poisons outside of the House of Black and White?Why did Nymeria leave Arya?Why did Arya not kill the Lannister soldiers she encountered in the Riverlands?What is the current canonical age of Sansa, Bran and Arya Stark?